Psych Flashcards

1
Q

Which one of the following is a predictor of a poor response to psychotherapy in adolescents with major depressive disorder? (check one)
-High global functioning on assessment
-Hypersomnia
-Inappropriate guilt
-Presence of family conflict

A

Presence of family conflict

Predictors of a poor response to psychotherapy in adolescents with major depressive disorder include presence of family conflict, severe depression, low global functioning on assessment, high scores on suicidality measures, coexisting anxiety, distorted thought patterns, and feelings of hopelessness.
High global functioning on assessment, hypersomnia, and inappropriate guilt are not predictors of a poor response to psychotherapy in adolescents with major depressive disorder.

How well did you know this?
1
Not at all
2
3
4
5
Perfectly
2
Q

Which one of the following is associated with the use of stimulant medications for attention-deficit disorder in adults? (check one)
- Lower success rates compared to nonstimulant medications
- Weight gain
- A low risk of medication abuse
- Serious adverse cardiovascular events
- Increases in blood pressure

A

Increase in BP

Stimulants are preferred over nonstimulant medications for adults with attention-deficit disorder. Stimulant medications can aggravate psychosis, tics, or hypertension and are therefore contraindicated in patients with these problems. The main side effects of these drugs include insomnia, dry mouth, weight loss, headaches, and anxiety. They are classified as schedule II drugs due to their potential for abuse. The risk for serious adverse cardiovascular events is very low, although these drugs can increase resting heart rate and elevate both systolic and diastolic blood pressure.

How well did you know this?
1
Not at all
2
3
4
5
Perfectly
3
Q

A 45-year-old male presents to your office accompanied by his sister. He tells you that 6 months ago he was laid off from a job where he had been employed for more than 20 years. He says that he declined a promotion 2 years ago because he was worried about working with a new team. He has been unable to secure a new job, stating that the interview process has been embarrassing and that he feels unfairly judged when his resume is critiqued. He reports feeling inadequate and says that he is more comfortable staying at home alone. His sister adds that he has been shy since he was a teenager, and now seems unwilling to change despite his dire financial situation. Depression and anxiety screenings are negative. He does not use alcohol or other substances.

Which one of the following personality disorders is most consistent with this patient’s presentation? (check one)
Antisocial
Avoidant
Borderline
Dependent
Histrionic

A

Avoidant

In order to make a diagnosis, personality disorders must meet specific criteria as outlined in the DSM-5. Other mental disorders, substance use or exposure, and medical conditions must also be excluded. This patient has avoidant personality disorder, which is characterized by social inhibition, fears of inadequacy, and hypersensitivity to criticism or rejection. It often presents in early adulthood. Persons with avoidant personality disorder may avoid new or unfamiliar situations, such as this patient who is unwilling to seek a new job. Persons with antisocial personality disorder exhibit a lack of respect for the rights of others, as well as deceitfulness, aggressiveness, and recklessness. Psychopathy and sociopathy are alternate terms. Borderline personality disorder is marked by instability in interpersonal relationships and self-image, impulsivity, reactivity of mood, and self-destructive behavior. Dependent personality disorder is described as an excessive need to be taken care of, intense fear of being alone, and extreme reliance on others for motivation and direction. Persons with histrionic personality disorder demonstrate excessive emotionality and attention-seeking behavior, often overestimating the closeness of interpersonal relationships and alienating others with hypersexual or hyperemotional reactions.

How well did you know this?
1
Not at all
2
3
4
5
Perfectly
4
Q

In the hospital setting, the use of atypical antipsychotics is most appropriate for which one of the following conditions? (check one)
Hospital-associated insomnia
ICU-associated delirium
Resistance to care in a patient with dementia
Aggression in a patient with dementia

A

ICU-associated delirium

Atypical antipsychotics may reduce the duration of delirium in adult intensive-care patients, and are recommended by the American College of Critical Care Medicine in their clinical practice guidelines for the management of pain, agitation, and delirium in adult patients in the intensive-care unit.

The American Geriatrics Society and the American Psychiatric Association (APA) recommend not using antipsychotics as a first choice to treat the behavioral and psychological symptoms associated with dementia, such as aggression and resistance to care. These drugs have limited benefit and can cause serious harm, including stroke and premature death. The APA also recommends against routinely prescribing two or more antipsychotic medications concurrently, and against routinely prescribing antipsychotic medications as a first-line intervention for insomnia in adults.

How well did you know this?
1
Not at all
2
3
4
5
Perfectly
5
Q

For patients on lithium monotherapy for bipolar disease, monitoring should include periodic blood levels of lithium, creatinine, and (check one)
calcium
hemoglobin A1c
lipids
testosterone
TSH

A

TSH

The concentration of lithium into the thyroid gland inhibits iodine uptake, iodotyrosine coupling, and thyroid hormone secretion. Thyroglobulin structure is also affected by lithium. The effect can be significant enough to produce a state of hypothyroidism and/or goiter, and studies have shown that as many as two-thirds of patients develop hypothyroidism within 10 years of beginning lithium treatment. Routine monitoring of TSH and T4 every 6–12 months is a recommended standard for all patients receiving lithium treatment. Lithium administration would not be expected to directly affect any of the other blood levels listed, so the indications for obtaining these tests are the same as for other patients.

How well did you know this?
1
Not at all
2
3
4
5
Perfectly
6
Q

A 24-year-old female complains of irritability, anxiety, and feeling restless. These symptoms began 3 months ago after she was in a car accident in which two people died. She has become very socially withdrawn and when she tries to sleep she has flashbacks to the accident.

In addition to recommending trauma-focused psychotherapy, which one of the following medications would be most appropriate? (check one)
Buspirone
Clonazepam (Klonopin)
Quetiapine (Seroquel)
Topiramate (Topamax)
Sertraline (Zoloft)

A

Sertraline (Zoloft)

Posttraumatic stress disorder (PTSD) occurs in approximately 20% of women and 8% of men exposed to traumatic events. Symptoms of PTSD include reexperiencing the event, depression, anxiety, changes in behavior, restlessness, social withdrawal, hypervigilance, poor attention, irritability, and fear. Many people with PTSD suffer from anxiety, depression, and substance abuse, and as many as one in five attempt suicide. Treatment with a combination of trauma-focused therapy and medications is recommended. SSRIs and SNRIs are considered first-line treatment. While paroxetine and sertraline are the only ones FDA-approved for PTSD, any of these drugs may be used. Other antidepressant medications can be used but are considered second-line treatment. Benzodiazepines have been used to treat the symptoms of hyperarousal but can worsen other PTSD symptoms and should be avoided. Studies of mood stabilizers in the treatment of PTSD have been mixed and many guidelines discourage their use. Antipsychotic medications are also not recommended. A large multi-site trial of risperidone reported no benefit over placebo.

How well did you know this?
1
Not at all
2
3
4
5
Perfectly
7
Q

An obese 70-year-old male with chronic pain due to osteoarthritis complains of fatigue, anhedonia, hypersomnolence, and increased appetite. Which one of the following would be the best pharmacologic agent for this patient? (check one)
Duloxetine (Cymbalta)
Mirtazapine (Remeron)
Citalopram (Celexa) D
Paroxetine (Paxil)
Nortriptyline (Pamelor)

A

Duloxetine (Cymbalta)

The best pharmacologic agent for this patient is duloxetine, as it is indicated for both depression and
chronic pain and is unlikely to cause weight gain. The other agents listed can cause weight gain to varying
degrees, and the tricyclic antidepressant nortriptyline is on the Beers list of drugs not recommended for
elderly patients (SOR A).

How well did you know this?
1
Not at all
2
3
4
5
Perfectly
8
Q

An 85-year-old male admitted to the hospital for shortness of breath is diagnosed with terminal lung cancer. He decides he would like to receive home hospice care. Over the course of his hospitalization he becomes increasingly confused and forgets where he is and why he is there. He appears depressed with a flat affect. He repeatedly tries to get out of bed and pulls at his IV line and catheter.

Which one of the following medications would be most appropriate for treating these symptoms? (check one)
Haloperidol
Nortriptyline (Pamelor)
Pentobarbital (Nembutal)
Lorazepam (Ativan)
Mirtazapine (Remeron)

A

Haloperidol

This patient is showing signs of delirium, which is common in hospice patients. Delirium should be considered in anyone with disturbances of cognitive function, altered attention, fluctuating consciousness, or acute agitation. The mainstay of management is the diagnosis and treatment of any conditions that may cause delirium. Medications that may cause delirium should be discontinued or reduced if possible. Antipsychotic medications are the drug of choice to improve delirium. Central nervous system depressants such as benzodiazepines and barbiturates should be avoided because they can make delirium worse. Nortriptyline has anticholinergic side effects and can also cause delirium. Mirtazapine would not be helpful for treating delirium.

How well did you know this?
1
Not at all
2
3
4
5
Perfectly
9
Q

You see a previously healthy 8-year-old female for a well child check. She was born at full term and adopted at birth. She has a history of methamphetamine exposure in utero. She is up to date on vaccinations and is doing well academically. She says she has friends at school and her mother confirms that her teachers report that she interacts well with the other students. Her mother notes, however, that the patient has persistent difficulties with anger and irritability. This behavior has been present since preschool, and while her mother thinks there has been some improvement, she is concerned that it has not resolved.

At home, the child has frequent outbursts, often speaks hatefully when upset, refuses to follow instructions, and throws herself on the floor and kicks in frustration at times. Her sisters sometimes worry about upsetting her because they know she will react dramatically, although she has not been physically aggressive. Her mother notes that the patient often blames her sisters or others when she misbehaves.

Which one of the following would be the most appropriate next step? (check one)
Reassurance that the behavior should continue to improve with age
Education on positive reinforcement of desired behaviors
Obtaining further history to evaluate for additional mental health conditions
A trial of risperidone (Risperdal)
Referral for parent management therapy

A

Obtaining further history to evaluate for additional mental health conditions

This child displays characteristics of oppositional defiant disorder (ODD). The DSM-5 criteria for a diagnosis of ODD include frequently losing one’s temper, being easily annoyed, antagonism toward authority figures, deliberately annoying others, placing blame on others, and being spiteful or vindictive. These symptoms must occur for at least 6 months, cause distress or negative impacts, and not occur exclusively with substance use or in the course of a psychotic, depressive, or bipolar disorder. Treatment of common comorbid mental health conditions can be associated with improvement in ODD, so it is important to evaluate for attention-deficit/hyperactivity disorder, depression, and anxiety disorders, as well as ODD.

Given the persistence of symptoms and maternal concern in this patient, reassurance alone would not be appropriate. Patients with ODD have a high risk of developing other mental health conditions later, and early therapy is recommended. While positive reinforcement is an important parenting strategy for children with ODD, it would not be expected to be effective in isolation. Medication is rarely indicated for ODD, and not as monotherapy. Parent management therapy is an important part of ODD treatment, but therapy should generally include both child therapy and parent training.

How well did you know this?
1
Not at all
2
3
4
5
Perfectly
10
Q

A 21-year-old female sees you because of a depressed mood since the birth of her son 2 months ago. She is breastfeeding, and her baby is doing well. She reports no difficulties sleeping, other than what is to be expected when caring for a newborn. She denies any suicidal or homicidal ideation and has never had thoughts about hurting the baby. She has a history of depression 2 years ago that was associated with starting college and feeling very isolated in the dormitory. She began taking sertraline (Zoloft), changed her schedule, and spent more time exercising. Within 6 months her depression resolved and she stopped the medication. She reports this current depression feels worse than her previous depression.

Which one of the following would be the most appropriate medication for this patient? (check one)
Amitriptyline
Diazepam (Valium)
Phenytoin
Sertraline
Zolpidem (Ambien)

A

Sertraline

SSRIs are the most commonly used medications for postpartum depression. They have fewer side effects and are considered safer than tricyclic antidepressants, especially in depressed women who may be at increased risk for medication overdose (SOR C). In one study, infant serum levels of sertraline and paroxetine were undetectable. It is also recommended that a woman with postpartum depression be started on a medication that she had taken previously with a good response, unless there is evidence of potential harm to her infant (SOR C).

Tricyclic antidepressants are excreted into breast milk and there is some concern regarding potential toxicity to the newborn. Phenytoin, diazepam, and zolpidem are not antidepressants. Phenytoin and diazepam are Category D for use in pregnant women. Diazepam is potentially toxic to infants and can accumulate in breastfed infants, and it is not recommended for lactating women (SOR C). Zolpidem is category B in pregnancy and probably acceptable for use in lactating women if clinically indicated.

How well did you know this?
1
Not at all
2
3
4
5
Perfectly
11
Q

A 15-year-old male is brought to the clinic for evaluation of his eating habits. His parents note that on several occasions they have found him alone with multiple empty food packages. He confirms that yesterday he consumed two fried chicken sandwiches, two orders of French fries, and two milkshakes in a 1-hour time period in which he was alone in his room. After this episode he was uncomfortably full for several hours.

Which one of the following would support your suspicion that this individual has binge-eating disorder? (check one)
Distorted body image
Feeling content after eating
Markedly low body weight for age and sex
Sense of loss of control during overeating episodes
Use of laxatives to control weight

A

Sense of loss of control during overeating episodes

Eating disorders are potentially life-threatening. The true prevalence of eating disorders is unclear, but it is estimated that 2%–4% of adolescents may meet criteria for binge-eating disorder (BED) with equal distribution across gender. The DSM-5 has specific criteria for diagnosing eating disorders. BED has several key features, including recurrent episodes of binge eating. Binge eating occurs in a distinct period of time and consists of consuming an amount of food outside the accepted norms for a meal, typically 3000–5000 kcal, and experiencing a sense of loss of control during the episode. Distorted body image and markedly low body weight for age and sex are seen in anorexia nervosa. There is no weight specification in the diagnostic criteria for BED and it is estimated that 50% of those suffering with BED are overweight or obese. Patients with BED feel a sense of distress, anguish, or despair (not contentment) after the eating episode and regarding this eating pattern in general. The use of laxatives to control weight or compensate for binge eating is typically seen with bulimia nervosa.

How well did you know this?
1
Not at all
2
3
4
5
Perfectly
12
Q

A 25-year-old female presents to your office for an annual health maintenance visit. You note a BMI of 17 kg/m2, a heart rate of 66 beats/min, and a blood pressure of 110/64 mm Hg. The patient reports exercising for 2 hours each day, incorporating cardio and light weights. The patient presents a detailed food diary and asks for advice about how to adjust her nutrition to lose weight.

In order to provide the best care for this patient, which one of the following would you order? (check one)
A fecal calprotectin level
Stool cultures
A DEXA scan
Chest radiography
Thyroid ultrasonography

A

A DEXA scan

This patient likely has an eating disorder. A DEXA scan is recommended to assess for low bone mineral density in patients with suspected or diagnosed eating disorders. Other appropriate screenings include orthostatic vital signs; a basic metabolic panel; a CBC; magnesium, phosphorus, prealbumin, and amylase levels; thyroid testing; and an EKG. Fecal calprotectin levels, stool cultures, chest radiography, and thyroid ultrasonography are not appropriate for this patient.

How well did you know this?
1
Not at all
2
3
4
5
Perfectly
13
Q

A 38-year-old female with a past medical history of posttraumatic stress disorder (PTSD) seeks treatment for recurring nightmares, which she has been experiencing for over 2 years following a near-drowning experience. During the daytime, she has unwanted intrusive thoughts and flash images of her drowning incident, and she avoids going near swimming pools. Her nightmares are increasingly distressing and she loses several hours of sleep nightly despite adequate behavioral therapy.

Which one of the following is the best initial pharmacotherapy for PTSD-associated nightmares? (check one)
Clonazepam (Klonopin)
Gabapentin (Neurontin), 300 mg daily
Prazosin (Minipress)
Sertraline (Zoloft)
Venlafaxine (Effexor XR)

A

Prazosin (Minipress)

Posttraumatic stress disorder (PTSD) is a psychiatric condition associated with previous exposure to a traumatic event (or events). There are four symptom categories: intrusive/re-experiencing (e.g., flashbacks, nightmares), avoidance/numbing, negative change in cognition and mood, and hyperarousal (e.g., anger outbursts, hypervigilance).

Prazosin, an α1-adrenergic receptor antagonist, is efficient and remains the first choice for pharmacologic therapy of PTSD-associated nightmares. The data is insufficient to support the use of clonazepam for treating nightmares associated with PTSD. Gabapentin and sertraline may be used as adjunctive therapy with antipsychotic and anxiolytic agents in treating PTSD-associated nightmares. Venlafaxine may improve behavioral symptoms, but it is ineffective for treating PTSD-associated nightmares.

How well did you know this?
1
Not at all
2
3
4
5
Perfectly
14
Q

Staff members in your practice often complain about one of your patients. He exhibits odd
behaviors and beliefs, and is always very anxious about his visit and about when he will be seen,
despite long familiarity with your practice.

Which one of the following personality disorders best fits the description of this patient? (check one)
Antisocial
Borderline
Dependent
Narcissistic
Schizotypal

A

Schizotypal

This patient most likely has schizotypal personality disorder. These patients have problems with social and
interpersonal relationships, which are marked by significant anxiety and discomfort, and they also exhibit
odd thinking, speech, and perceptions. This disorder is classified as being in the cluster A personality
disorder group. Patients with disorders in this group exhibit odd or eccentric personalities, and the group
includes paranoid, schizoid, and schizotypal personality disorders.
Cluster B disorders are characterized by dramatic, emotional, or erratic personalities, and include
antisocial, borderline, histrionic, and narcissistic personality disorders. Cluster C disorders include
avoidant, obsessive-compulsive, and dependent personality disorders. Patients with disorders in this group
exhibit mainly anxious or fearful behaviors.

How well did you know this?
1
Not at all
2
3
4
5
Perfectly
15
Q

You have diagnosed chronic fatigue syndrome in a 32-year-old female. Her PHQ-9 is negative for depression. An evaluation for sleep disturbance and other comorbid disorders is also negative.

Which one of the following would be the most effective treatment? (check one)
Cognitive-behavioral therapy
Interpersonal therapy
Citalopram (Celexa)
Methylphenidate (Ritalin)

A

Cognitive-behavioral therapy

The criteria for chronic fatigue syndrome include fatigue for 6 months and a minimum of four of the following physical symptoms: impaired memory, postexertional malaise, muscle pain, polyarthralgia, tender lymph nodes, sore throat, new headaches, and unrefreshing sleep. Both cognitive-behavioral therapy and graded exercise therapy have been shown to improve fatigue levels, anxiety, work/social adjustment, and postexertional malaise (SOR A). Treatments that have not been shown to be effective include methylphenidate, melatonin, and galantamine. Citalopram has not been shown to be effective in the absence of a comorbid diagnosis of depression.

How well did you know this?
1
Not at all
2
3
4
5
Perfectly
16
Q

A 32-year-old female has a 3-week history of depressed mood. She reports markedly diminished interest or pleasure in most activities, fatigue, a diminished ability to concentrate, and insomnia. She has had recurrent suicidal thoughts, but has no specific plan. Further investigation reveals a past history of several hypomanic episodes lasting 4–5 days, characterized by a persistently elevated, expansive mood. During these episodes she needed little sleep, was talkative, met multiple goals, and had trouble keeping up with the thoughts that were running through her head. She was treated with lithium in her early twenties but she stopped taking it because it stifled her artistic creativity. She currently takes no medication.

Her physical examination is unremarkable. Results from comprehensive laboratory studies, including a urine toxicology screen, are also normal.

Which one of the following is most appropriate for her current depressive symptoms? (check one)
Aripiprazole (Abilify)
Venlafaxine
Divalproex (Depakote)
Divalproex and bupropion (Wellbutrin)
Lithium and paroxetine (Paxil)

A

Divalproex (Depakote)

This patient has bipolar II disorder. She has a history of hypomanic episodes as well as major depression, with no history of a manic or mixed episode. Among the pharmacologic options listed, only divalproex and lithium are indicated for treating bipolar depression or acute mania, and for maintenance. They should be given as single agents, however, not in combination with other drugs. No evidence supports combination therapy or the addition of an antidepressant in the acute phase of depression.

In a study of patients with bipolar II disorder, initially adding paroxetine or bupropion to the mood stabilizer was no more effective than using lithium or valproate. An SSRI or bupropion can be added if a therapeutic dosage of a mood stabilizer does not resolve symptoms and the patient is not in a mixed state. Tricyclic antidepressants and antidepressants with dual properties, such as venlafaxine, should be avoided because they may induce mania. Aripiprazole is indicated for acute mania but not for bipolar depression.

How well did you know this?
1
Not at all
2
3
4
5
Perfectly
17
Q

The parents of a 5-year-old male ask you about treating him for attention-deficit/hyperactivity disorder (ADHD) because of his hyperactivity at home and preschool. According to the newest guidelines, the most appropriate next step is to? (check one)
prescribe a very low dose of stimulant medication
explain to the parents that drug therapy for ADHD is not appropriate at this age
perform a dietary history focusing on the child’s sugar intake
explore the nature of his hyperactivity and whether there are coexisting behavioral problems

A

explore the nature of his hyperactivity and whether there are coexisting behavioral problems

Guidelines from the American Academy of Pediatrics state that stimulant medication can be prescribed for preschool children, but only after a thorough trial of behavior modification. Foods and additives have never been shown to cause or aggravate ADHD. Children with ADHD often have other behavioral problems such as depression or oppositional-defiant disorder.

How well did you know this?
1
Not at all
2
3
4
5
Perfectly
18
Q

A patient who has terminal metastatic lung cancer with bony metastases is being cared for at home and using hospice services. The hospice nurse calls you during the night because the family had called her to come to the house. When she arrived she found the patient acutely agitated, confused, and disoriented, and he does not recognize his family members. The patient is trying to hit his caretakers, who are distressed by the situation.

In addition to checking for underlying causes of these acute symptoms, which one of the following is most appropriate for managing this problem?
(check one)
Amitriptyline
Haloperidol
Scopolamine
Trazodone (Oleptro)

A

Haloperidol

This patient is experiencing delirium, which is common in the last weeks of life, occurring in 26%–44% of persons hospitalized with advanced cancer and in up to 88% of persons with a terminal illness. In studies of a palliative care population it was possible to determine a cause for delirium in less than 50% of cases. There is a consensus based on observational evidence and experience that antipsychotic agents such as haloperidol are effective for the management of delirium, and they are widely used. However, there have been few randomized, controlled trials to assess their effectiveness.

While benzodiazepines are used extensively in persons with delirium who are terminally ill, there is no evidence from well-conducted trials that they are beneficial. Trazodone is an antidepressant that is sometimes used for insomnia. Scopolamine is an anticholinergic that is used to reduce respiratory secretions in hospice patients, but its anticholinergic side effects would increase delirium severity. Amitriptyline also has significant anticholinergic properties.

How well did you know this?
1
Not at all
2
3
4
5
Perfectly
19
Q

Which one of the following medications used to treat psychiatric disorders is associated with an increased risk of agranulocytosis?
(check one)
Carbamazepine (Tegretol)
Lithium
Aripiprazole (Abilify)
Olanzapine (Zyprexa)
Imipramine (Tofranil)

A

Carbamazepine (Tegretol)

People taking carbamazepine have a five-to eightfold increased risk of developing agranulocytosis. Baseline values including a CBC, serum electrolytes, and liver enzymes should be obtained before the drug is started, and the patient should be monitored with periodic hematologic testing. The other medications listed are not associated with agranulocytosis. Aripiprazole and olanzapine carry black box warnings for an increased risk of death in the elderly. Lithium is associated with lithium toxicity and thyroid dysfunction. Imipramine carries a warning for cardiac toxicity, and EKG monitoring is recommended.

How well did you know this?
1
Not at all
2
3
4
5
Perfectly
20
Q

An 87-year-old female is brought to the emergency department after losing consciousness at the dinner table. Her history indicates recent unintentional weight loss. Further evaluation ultimately reveals a large mass at the head of the pancreas and extensive metastasis to numerous organs, including the brain. Her life expectancy is estimated to be 2–3 weeks. The patient chooses to receive hospice care but becomes very depressed.

Which one of the following would be best for improving her depression?
(check one)
Electroconvulsive therapy
Methylphenidate (Ritalin)
Mirtazapine (Remeron)
Fluoxetine (Prozac)
Nortriptyline (Pamelor)

A

Methylphenidate (Ritalin)

There is good evidence that psychostimulants reduce symptoms of depression within days, making methylphenidate a good choice for this patient (SOR B). Electroconvulsive therapy is contraindicated due to her brain lesions. Mirtazapine, fluoxetine, and nortriptyline all take at least 3–4 weeks to have any antidepressant effects, and would not be appropriate given the patient’s life expectancy (SOR B).

How well did you know this?
1
Not at all
2
3
4
5
Perfectly
21
Q

The diagnosis of delirium is based on?

(check one)
The history and physical findings
Complete metabolic panel results
Toxicology screening results
EEG findings
MRI of the brain

A

The history and physical findings

The diagnosis of delirium is based entirely on the history and physical examination. No laboratory tests, imaging studies, or other tests are more accurate than clinical assessment.

How well did you know this?
1
Not at all
2
3
4
5
Perfectly
22
Q

A young adult who has been one of your patients for several years begins to exhibit symptoms of a thought disorder, and you are concerned about schizophrenia. In a review of the diagnostic criteria for schizophrenia, your resources refer to positive and negative symptoms.

Which one of the following is a negative symptom associated with schizophrenia? (check one)
Delusions
Depression
Disorganized speech
Hallucinations
Reduced speech

A

Reduced speech

The diagnosis of schizophrenia requires that at least one positive and one negative symptom or disorganized behavior be present for 6 months and be severe for at least 1 month. Positive symptoms include delusions, hallucinations, and disorganized speech. Negative symptoms include alogia (reduced number of words spoken), blunted affect, avolition (a decrease in motivated, self-initiated, purposeful activities), asociality, and anhedonia. Depression may be present in schizophrenia but is neither a positive nor negative symptom.

How well did you know this?
1
Not at all
2
3
4
5
Perfectly
23
Q

A 42-year-old female is troubled by her lack of interest in sex. She is generally healthy, takes no medications, and has regular menstrual periods. She is content with the emotional intimacy of her marriage and has had satisfying sexual interactions in the past. She does not have any religious or cultural barriers regarding her sexuality, and asks for ideas on how to improve her situation.

Which one of the following has consistent evidence of benefit in cases such as this?
(check one)
Cognitive-behavioral therapy
Viewing pornography
Oral estrogen
Oral sildenafil (Viagra)
Topical testosterone

A

Topical testosterone

This patient meets the criteria for hypoactive sexual desire disorder (HSDD). The incidence of this condition is variable based on the age, life stage, and culture of the patient, but is estimated to be present in about 5%–15% of the adult female population. This diagnosis includes two components: (1) recurrent deficiency or absence of sexual desire or receptivity to sexual activity, and (2) distress about such a deficiency. In menstruating women, oral estrogen and oral sildenafil have not been shown to be superior to placebo. Cognitive-behavioral therapy has been shown to be helpful for other sexual dysfunctions, but not with HSDD. Topical testosterone, in either patch or gel form, has shown consistent improvements in arousal, desire, fantasy, orgasm, and overall satisfaction in cases of HSDD.

How well did you know this?
1
Not at all
2
3
4
5
Perfectly
24
Q

Which one of the following has good evidence of effectively improving borderline personality disorder?
(check one)
SSRIs
Second-generation antipsychotics
Omega-3 fatty acids
No currently available pharmacotherapy

A

No currently available pharmacotherapy

There are no proven therapies to reduce the severity of borderline personality disorder (SOR A). The most promising psychological therapy is dialectic behavioral therapy (DBT). DBT is a multi-faceted program specifically designed to treat borderline personality disorder. The few, small studies of DBT found improvement in many symptoms of borderline personality disorder, but long-term data is lacking. Another promising therapy is psychoanalytic-oriented day hospital therapy. Again, study sizes have been small and data cannot be extrapolated to the population as a whole.
Omega-3 fatty acids, second-generation antipsychotics, and mood stabilizers have been shown to be helpful for some symptoms of borderline personality disorder but not for overall severity. Their benefits are based on single-study results and side effects were not addressed in the studies. SSRIs are not recommended for borderline personality disorder unless there is a concomitant mood disorder.

How well did you know this?
1
Not at all
2
3
4
5
Perfectly
25
Q

A 13-year-old female is brought to your office for evaluation of school difficulties and depressed mood. Her mother and older sister have both been diagnosed with depression. After a thorough history and physical examination, you diagnose major depressive disorder. You arrange for the patient to receive cognitive-behavioral therapy, but after 6 weeks her condition is only minimally improved.

Which one of the following medications would be appropriate to add to this patient’s treatment plan at this point?
(check one)
Fluoxetine (Prozac)
Imipramine (Tofranil)
Lithium
Venlafaxine

A

Fluoxetine (Prozac)

The diagnostic criteria for depression are the same for children and adults, although the manner in which these symptoms present may be different. Adolescents with depression are more likely to experience anhedonia, boredom, hopelessness, hypersomnia, weight change, alcohol or drug use, and suicide attempts. Psychotherapy should always be included as part of a treatment plan for depression in adolescents.

Cognitive-behavioral therapy and interpersonal therapy are two modalities that have been proven effective in the treatment of adolescent depression. Medications should be considered for more severe depression or depression failing to respond to psychotherapy. A Cochrane review found that fluoxetine was the only agent with consistent evidence of effectiveness in decreasing depressive symptoms in adolescents. Consensus guidelines recommend fluoxetine, citalopram, or sertraline as first-line treatments for moderate to severe depression in children and adolescents. Escitalopram is also licensed for the treatment of depression in adolescents age 12 or older. All antidepressants have a boxed warning regarding an increased risk of suicide; therefore, close monitoring is recommended to assess for suicidality and other adverse effects, such as gastrointestinal effects, nervousness, headache, and restlessness. Tricyclic antidepressants were previously used to treat depression in children, but studies have shown little to no benefit in adolescents and children.

How well did you know this?
1
Not at all
2
3
4
5
Perfectly
26
Q

A 60-year-old male smoker has lung cancer, and a life expectancy of 4–6 months. His wife is concerned about his state of mind and requests medication for him. His cancer-related pain is generally controlled.

When evaluating the patient, which one of the following features would be more characteristic of depression as opposed to a grief reaction?
(check one)
Insomnia
Loss of interest or pleasure in all activities
Feelings of guilt
Thoughts of wanting to die
Psychomotor agitation

A

Loss of interest or pleasure in all activities

While there is significant overlap in the symptoms of each condition, there are some signs and symptoms that help the family physician determine whether a terminally ill patient is experiencing grief or has major depression. This distinction is important because the terminally ill patient with depression would likely benefit from antidepressant medication, whereas a patient with end-of-life grief is generally best managed without psychotherapeutic medications.

The key clinical feature in distinguishing the two conditions is in the pervasiveness of symptoms in depression, particularly the loss of pleasure or interest in all activities. Episodic feelings of guilt, anxiety, and helplessness, and even thoughts of wanting to die can and do occur with grief reactions, but these feelings are not constant and over time the symptoms gradually wane. Terminally ill patients with major depression feel helplessly hopeless all the time, but they often respond to and significantly benefit from antidepressant medication (SOR A).

How well did you know this?
1
Not at all
2
3
4
5
Perfectly
27
Q

A 44-year-old female is brought to your office by her mother. The patient was in a severe car
accident 2 weeks ago. Her husband was killed instantly and she was extracted by emergency
responders almost an hour later. She received a full examination at a local emergency
department and was discharged home with only minor contusions and abrasions and no evidence
of a closed head injury.
The patient has been panicked and unable to sleep. She has recurrent flashbacks of the event and
dreams repeatedly about her husband’s death. She says that sometimes, even while awake, she
can almost sense her husband’s lifeless body near her. She has refused to get into a car since the
accident, which is the reason she has not sought care sooner. She has not been able to focus on
daily tasks but has been able to eat and drink adequate amounts.

Which one of the following diagnoses best describes her condition? (check one)
Acute stress disorder
Major depressive disorder
Obsessive-compulsive disorder
Panic disorder
Generalized anxiety disorder

A

Acute stress disorder

Acute stress disorder (ASD) lies on a spectrum of trauma-related disorders between adjustment disorder
and posttraumatic stress disorder (PTSD). ASD is differentiated from PTSD primarily by duration, with
PTSD requiring the presence of similar symptoms (intrusion, negative mood, dissociation, avoidance, and
arousal) for longer than 1 month. Conversely, adjustment disorder is a less severe condition than ASD that
involves either a less traumatic or threatening inciting event and/or less severe symptoms that do not meet
DSM-5 criteria for acute stress disorder.

How well did you know this?
1
Not at all
2
3
4
5
Perfectly
28
Q

Which one of the following cardiac rhythm abnormalities is most common in patients with anorexia nervosa?

(check one)
Atrial fibrillation
Ventricular fibrillation
Sinus bradycardia
Sinus tachycardia
Paroxysmal supraventricular tachycardia

A

Sinus bradycardia

Sinus bradycardia is almost universally present in patients with anorexia nervosa. It is hypothesized that this is due to vagal hyperactivity resulting from an attempt to decrease the amount of cardiac work by reducing cardiac output. It is also possible that the bradycardia can be accounted for by low serum T3 levels, a common finding in persons with chronic malnutrition. Sinus tachycardia may occur with refeeding in patients with anorexia. Other arrhythmias may also occur but are less frequent.

How well did you know this?
1
Not at all
2
3
4
5
Perfectly
29
Q

Which one of the following is most likely to induce withdrawal symptoms if discontinued abruptly? (check one)
Venlafaxine (Effexor)
Divalproex (Depakote)
Fluoxetine (Prozac)
Olanzapine (Zyprexa)
Donepezil (Aricept)

A

Venlafaxine (Effexor)

The abrupt discontinuation of venlafaxine, or a reduction in dosage, is associated with withdrawal symptoms much more severe than those seen with other SSRIs such as fluoxetine. Although more pronounced with higher dosages and prolonged administration, they also occur at lower dosages. These symptoms include agitation, anorexia, confusion, impaired coordination, seizures, sweating, tremor, and vomiting. To avoid this withdrawal symptom, dosage changes should be instituted gradually. Abrupt discontinuation of mood stabilizers such as divalproex, and atypical antipsychotics such as olanzapine, can result in the return of psychiatric symptoms, but not severe physiologic dysfunction. Similarly, stopping anticholinesterase inhibitors such as donepezil will not cause a withdrawal syndrome.

How well did you know this?
1
Not at all
2
3
4
5
Perfectly
30
Q

The most appropriate initial pharmacologic treatment of panic disorder is: (check one)
An SSRI
A tricyclic antidepressant
Valproic acid (Depakene)
Lithium

A

An SSRI

An SSRI is the treatment of choice for patients who have never had pharmacotherapy for panic disorder.

How well did you know this?
1
Not at all
2
3
4
5
Perfectly
31
Q

Which one of the following statements regarding antidepressant drug therapy is true? (check one)
The response rate to most antidepressants is 90%–95%
Patients unimproved after 2 weeks should receive a different drug
Patients unresponsive after 6 weeks should have their treatment altered
Patients unresponsive to one class of drugs are unlikely to respond to another class
In patients who have not improved after 6 weeks of drug therapy, depression is unlikely to be the cause of their symptoms

A

Patients unresponsive after 6 weeks should have their treatment altered

An adequate trial of antidepressant therapy is 4–6 weeks. Patients who are unresponsive to treatment may respond to another antidepressant with a different mechanism of action. Patients who are partially responsive may benefit from dosage titration or the addition of a second antidepressant in combination. Electroconvulsive therapy is the most effective treatment in patients with severe resistance to medical antidepressant therapy or those with psychotic depression.

How well did you know this?
1
Not at all
2
3
4
5
Perfectly
32
Q

Which one of the following is the most appropriate adjunct medication for treating patients with post-traumatic stress disorder? (check one)
Alprazolam (Xanax)
Haloperidol (Haldol)
Methylphenidate (Ritalin)
Sertraline (Zoloft)
Temazepam (Restoril)

A

Sertraline (Zoloft)

Results of randomized clinical trials demonstrate that medications such as SSRIs, tricyclic antidepressants, and monoamine oxidase inhibitors alleviate the symptoms of post-traumatic stress disorder (PTSD) and are associated with improvements in overall functioning. SSRIs are a first-line medication because they are safer and better tolerated than other types of psychotropic medications. Sertraline and paroxetine are the only agents that have been approved by the FDA for the treatment of PTSD.

How well did you know this?
1
Not at all
2
3
4
5
Perfectly
33
Q

A 9-year-old male has persistent severe depression despite cognitive behavioral therapy. Which one of the following medications is approved by the FDA for the treatment of major depressive disorder in this age group? (check one)
No antidepressant medications
Bupropion (Wellbutrin)
Escitalopram (Lexapro)
Fluoxetine (Prozac)
Sertraline (Zoloft)

A

Fluoxetine (Prozac)

Only two medications, escitalopram and fluoxetine, are approved by the FDA for the treatment of major depressive disorder in children and adolescents. Fluoxetine is approved for treatment in children age≥8 while escitalopram is only approved for use in children age ≥12. Bupropion and sertraline are not approved by the FDA for the treatment of major depressive disorder in children.

34
Q

Your patient, who is overweight and says she has struggled with bulimia in the past, asks for your advice on strategies to help prevent obesity and eating disorders in her 12-year-old daughter. Which one of the following strategies should be incorporated? (check one)
Implementing a diet of moderate calorie restriction for healthy adolescents and adults 1 week per month
Having more frequent discussions regarding weight control and healthy eating
Limiting home-prepared dinners to 1–2 times per week
Eating meals together as a family ³7 times per week
Watching television during mealtimes

A

Eating meals together as a family ³7 times per week

A higher frequency of family meals is associated with improved dietary quality, as evidenced by increased consumption of fruits, vegetables, and grains. Eating meals together as a family on most days or every day is protective against purging and binge eating, as well as frequent dieting, which is a risk factor for both obesity and eating disorders. Parental talk about weight revolving around their children or their own dieting is linked to becoming overweight. Meals that are home-prepared and undistracted are also beneficial to maintaining healthy weight and attitudes toward food.

35
Q

An 18-year-old female with a BMI of 21 kg/m2 sees you for evaluation of recent hair loss and amenorrhea. During the review of systems she tells you that she thinks she is fat and reveals that she recently went on a restrictive diet and lost 15% of her body weight. On examination she is ill appearing, and her affect is blunted. Her blood pressure is 90/60 mm Hg and her heart rate is 50 beats/min. Laboratory testing reveals euthyroid sick syndrome and a decreased serum calcium level. Bone mineral density (BMD) testing shows bone loss. You refer her for extensive psychologic intervention.

Once this patient begins showing signs of improvement, the best indication for recovery of BMD would be (check one)
increased food intake
a normal calcium level
normal thyroid studies
the return of menses
weight restoration

A

the return of menses

This patient has anorexia nervosa. While weight restoration is important, the return of menses is the best indication of bone mineral density (BMD) recovery. Increased food intake, a normal calcium level, and normal thyroid function are important but not indicative of BMD recovery.

36
Q

A 23-year-old graduate student presents to establish care at your office with a 2-week history of symptoms consistent with a major depressive episode. She reports being given a diagnosis of unipolar depression at age 16. For several years, she managed her symptoms effectively with psychotherapy alone. Last year, she was hospitalized for an initial episode of mania with psychotic features and was treated successfully with lithium and risperidone (Risperdal). After returning to her graduate studies 1 month later, she maintained a 9-month period of euthymia on lithium monotherapy before her recent major depressive symptoms began. She does not have any history of recreational drug use. A physical examination, CBC, comprehensive metabolic panel, and TSH level are all normal, and a urine pregnancy test is negative.

According to the DSM-5, her mood disorder diagnosis is (check one)
bipolar I disorder
bipolar II disorder
bipolar disorder, not otherwise specified
cyclothymia
schizoaffective disorder

A

bipolar I disorder

The primary feature that distinguishes bipolar I disorder from bipolar II disorder is a history of a manic or a mixed manic/depressive episode. Additionally, in bipolar I, there may or may not be a history of psychosis and/or major depression. This patient has a history of a manic episode with psychotic features involving hospitalization, which fulfills the criteria for bipolar I disorder. Her major depressive episodes are consistent with a diagnosis of bipolar I but are not essential to it.

Patients with bipolar II disorder, in contrast, have a history of hypomania along with major depression, although no history of mania. Cyclothymia includes hypomanic and depressive symptoms that do not meet bipolar II criteria or a major depressive episode and occur over 2 or more years with no more than 2 symptom-free months during that time. Bipolar disorder, not otherwise specified, defines cases that do not meet full criteria for bipolar I, bipolar II, or cyclothymia, for example, in the setting of manic symptoms lasting less than 1 week, without psychosis or hospitalization. Although overlap exists between bipolar and schizoaffective disorders, for the latter, patients must meet criteria for schizophrenia, which includes a minimum of 2 weeks of euthymic psychosis.

37
Q

A 36-year-old female has been seen multiple times in the past several months for various pain-related complaints. On each occasion, no physical or laboratory findings were found to explain the symptoms. The patient is involved in a workers compensation case and could make a significant amount of money if it is demonstrated that her physical complaints are related to work conditions. Which one of the following diagnoses characterizes her unexplained physical symptoms? (check one)
Somatization disorder
Conversion disorder
Hypochondriasis
Malingering

A

Malingering

This patient most likely is malingering, which is to purposefully feign physical symptoms for external gain. Factitious disorder involves adopting physical symptoms for unconscious internal gain, such as deriving comfort from taking on the role of being sick. Somatization disorder is related to numerous unexplained physical symptoms that last for several years and typically begin before 30 years of age. Conversion disorder involves a single voluntary motor or sensory dysfunction suggestive of a neurologic condition, but not conforming to any known anatomic pathways or physiologic mechanisms.

38
Q

A 35-year-old female sees you because she has lost her voice. She has had no recent upper respiratory infection symptoms, cough, or heartburn, and she has not done anything that would strain her voice. Findings are normal on examination of the head and neck. A review of her chart shows this has happened before, but an ear, nose, and throat evaluation found no abnormalities. She also has been seen numerous times in the past few years for headaches, chest pains, abdominal pains, rectal pressure, and vaginal symptoms. Despite several workups and referrals, no definite cause has been found and the symptoms persist. Which one of the following would be the most reasonable plan of action? (check one)
Test for food allergies
Begin low-dose lorazepam (Ativan)
Begin a 6-week trial of a proton pump inhibitor
Schedule frequent office visits

A

Schedule frequent office visits

Somatization disorders should be considered in patients who have a history of various complaints over a several-year period that involve multiple organ systems. There is no test to confirm this diagnosis. It is often intertwined in other psychiatric problems, including anxiety disorder, personality disorder, and depression. Treatment includes testing to make sure that there is nothing physically wrong, while building a trusting relationship with the patient. Once this is accomplished, it is reasonable to discuss the disorder with the patient. Cognitive therapy has been shown to be of value, as well as regularly scheduled office visits for monitoring and support. Medicines for coexisting psychiatric problems also are of benefit. In addition, referral for psychiatric consultation may be worthwhile. Food allergies can cause a variety of symptoms, but usually not to the extent seen with this patient, and testing for this might confuse the issue. Lorazepam may help the symptoms if there is a coexisting anxiety disorder, but it will not address the underlying problem. Laryngeal esophageal reflux can cause hoarseness and will respond to proton pump inhibitors, but given the repetitive nature of her symptoms and the previous negative workups, it is not consistent with the whole picture.

39
Q

A 64-year-old white male appears to be depressed 2 weeks after hospital discharge for a myocardial infarction. He experienced short runs of ventricular tachycardia during his hospitalization, and echocardiography revealed an ejection fraction of 40% at the time of discharge, with no symptoms of heart failure. He has a history of depression in the past. His current symptoms include depressed mood, sleep disturbance, feelings of hopelessness, and anhedonia. He denies suicidal ideation.Which one of the following would be most appropriate at this point? (check one)
Low-dose amitriptyline at bedtime
Sertraline (Zoloft)
Referral for electroconvulsive therapy
Referral for intense interpersonal psychotherapy

A

Sertraline (Zoloft)

Several studies have demonstrated that SSRIs are safe and effective in treating depression in patients with coronary disease, particularly those with a history of previous episodes of depression. Medications have performed significantly better than intensive interpersonal psychotherapy in this setting. Electroconvulsive therapy is not considered first-line therapy in the absence of severe symptoms. While it may be effective for sleep disturbance, amitriptyline has potential cardiac side effects and is unlikely to be effective for the treatment of depression in low doses.

40
Q

A 32-year-old female presents to your office 3 months after surviving a serious rollover car accident. Since the accident she has had flashbacks, nightmares, and difficulty sleeping. She has been unable to resume work or care for her young children due to difficulty concentrating and feeling like she is in a daze. She has not been able to drive, and riding in a vehicle triggers anxiety and fear. She tells you that she cannot stop feeling responsible for the accident. She does not take any medications and has no history of substance use. After performing a structured diagnostic interview and review of DSM-5 criteria to confirm your diagnosis, you discuss treatment options. She is not willing to consider psychotherapy at this time.

Which one of the following would be the most appropriate pharmacotherapy? (check one)
Clonazepam (Klonopin)
Divalproex (Depakote)
Fluoxetine (Prozac)
Quetiapine (Seroquel)
Risperidone (Risperdal)

A

Fluoxetine (Prozac)

This patient has posttraumatic stress disorder (PTSD). She was exposed to threatened death and injury (DSM-5 criterion A) and exhibits multiple symptoms from several clusters of the DSM-5 criteria for PTSD (reliving of the traumatic event [criterion B], avoidance of trauma-related stimuli [criterion C], negative thoughts or feelings that began or worsened after the trauma [criterion D], and trauma-related arousal and reactivity that began or worsened after the trauma [criterion E]). She has symptoms that have caused distress and functional impairment for more than 1 month and are not triggered by medication or substance use (criteria F–H). Individual, trauma-focused psychotherapy has strong evidence for benefit in the treatment of PTSD and is recommended as the first-line treatment. If psychotherapy is not available or preferred by the patient, pharmacotherapy is then recommended. Among the options listed, fluoxetine has the strongest evidence of efficacy as monotherapy for PTSD. There is a lack of evidence for the efficacy of benzodiazepines such as clonazepam, antiepileptics such as divalproex, and atypical antipsychotics such as quetiapine and risperidone. Furthermore, risks outweigh any potential benefits from these medications.

41
Q

A 34-year-old female presents to your office and insists that she be seen. She seeks your care frequently, showering you with compliments. Today she says that she feels depressed and hurts “all over.” She reports that your medical assistant is not being friendly to her. She states that her symptoms started a few days ago, after her boyfriend broke up with her. She had thought he would propose soon although they had been dating for 2 months. She says he must have left her because she is unattractive. She asks to be tested for sexually transmitted infections since she frequently engages in unprotected intercourse. She was recently pulled over by a police officer for reckless driving, but states she was being punished unfairly.

A physical examination reveals a BMI of 31 kg/m2, stable vital signs, and several scars on her upper arms, but is otherwise unremarkable. She appears anxious and depressed, and as in the past, mentions suicidal thoughts. Your staff asks you if she is just a difficult patient or whether a disease process is involved.

Which one of the following is the most likely diagnosis for this patient? (check one)
Bipolar disorder
Borderline personality disorder
Generalized anxiety disorder
Major depressive disorder
Posttraumatic stress disorder

A

Borderline personality disorder

Borderline personality disorder is characterized by pervasive patterns of instability that affect regulation, self-image, and interpersonal relationships. These patients may frequently seek medical care, sabotage healthy behavior, and engage in high-risk sexual behaviors and substance use. These patients also have an increased suicide risk. The hallmark symptoms are unstable interpersonal relationships, chronic suicidal tendencies, a negative self-image, difficulty controlling moods, and marked impulsivity. Patients with borderline personality disorder can also present with vague somatic complaints and chronic pain. It is thought to be caused by a combination of neurobiologic, psychosocial, and genetic factors. Treatment includes a multifaceted approach with psychotherapy being the first-line treatment, but patients do have a high dropout rate. There are no FDA-approved medications for the treatment of borderline personality disorder, although SSRIs and quetiapine are commonly prescribed.

Bipolar disorder is characterized by alternating periods of depression and mania or, in the case of bipolar type II, hypomania. The DSM-5 criteria for generalized anxiety disorder include excessive worry and anxiety about many events, difficulty controlling the worry, and three or more of the following symptoms: restlessness, fatigue, concentration difficulty, irritability, muscle tension, and sleep disturbance.

The essential feature of posttraumatic stress disorder is the development of characteristic symptoms after experiencing a psychologically traumatic event (or events) outside the range of human experience usually considered to be normal. The characteristic symptoms involve reexperiencing the traumatic event; numbing of responsiveness to, or involvement with, the external world; and a variety of other autonomic, dysphoric, or cognitive symptoms, such as exaggerated startle response, difficulty concentrating, memory impairment, feelings of guilt, and sleep difficulties. Criteria for major depressive disorder include depressed mood, loss of interest or pleasure, appetite or weight changes, sleep disturbances, fatigue, and difficulty concentrating.

42
Q

A 20-year-old female with a history of persistent depression sees you for follow-up. She reports unplanned weight loss, chronic insomnia, fatigue, a loss of interest in hobbies that she previously enjoyed, and decreased concentration. She has seen other physicians and has tried antidepressant medications, including paroxetine (Paxil), bupropion (Wellbutrin), and citalopram (Celexa). She reports that there was a week when her mood felt much more normal. She needed only about 3 hours of sleep each night during that week. She admits she was somewhat impulsive, however, and experienced racing thoughts. She does not abuse any substances but has several family members who are drug abusers. Her mother and older sister committed suicide. Her examination, laboratory work, and drug screen are all unremarkable.

Which one of the following is most likely to help this patient? (check one)
Escitalopram (Lexapro)
Mirtazapine (Remeron)
Quetiapine (Seroquel)
Trazodone (Oleptro)
Venlafaxine (Effexor XR)

A

Quetiapine (Seroquel)

Bipolar disorders often present in late childhood or early adolescence. Outcomes can be improved by early recognition. Manic episodes that occur with bipolar I disorder are usually easy to identify. However, patients with bipolar II disorder may have a hypomanic episode that goes unrecognized, and the patient may present with persistent depression. There is usually a family history of bipolar disorder or multiple relatives having persistent depression, obsessive-compulsive disorder, attention-deficit disorder, or panic disorder. There may be multiple instances in the family of suicide, drug abuse, alcohol abuse, or incarceration. The patient has likely failed to respond to at least three antidepressant drugs and may have a history of multiple divorces.

There are options for the treatment of bipolar depression. Quetiapine usually leads to a response after 1 week of therapy but is associated with weight gain and extrapyramidal side effects. Olanzapine may also be used but should be combined with an SSRI. Lithium may also be effective for acute depression. Lamotrigine is effective but titration should be spread over at least 6 weeks in order to decrease the risk of Stevens-Johnson syndrome.

43
Q

A 27-year-old male with a diagnosis of depression prefers to avoid pharmacologic treatment. You agree to engage in a trial of therapy in your office. During the treatment process, you help the patient realize that some of his perceptions and interpretations of reality may be false and lead to negative thoughts. Next, you help him discover alternative thoughts that reflect reality more closely, and to learn to discard his previous distorted thinking. By learning to substitute healthy thoughts for negative thoughts, he finds his mood, behavior, and physical reaction to different situations are improved.
Which one of the following best categorizes this type of therapy?
(check one)
Psychoanalysis
Biofeedback
Cognitive therapy
Group psychotherapy
Hypnosis therapy

A

Cognitive therapy

This patient is engaged in cognitive therapy, which is a treatment process that helps patients correct false self-beliefs that can lead to negative moods and behaviors. Cognitive therapy has been shown to effectively treat patients with unipolar major depression, and is particularly useful in patients who do not respond to medication or who prefer nonpharmacologic therapy.
Psychoanalysis is a process of free association where repressed memories are recovered. Biofeedback involves instrumentation that gives feedback about a patient’s physiologic response to various situations in order to bring the autonomic nervous system under voluntary control. Group psychotherapy is a form of treatment in which people who are emotionally ill meet in a group guided by a trained therapist and help one another effect personality change. Hypnosis involves helping a patient enter a state of heightened focal concentration and receptivity that is typified by a feeling of involuntariness or an altered state of consciousness.

44
Q

Which one of the following is associated with an increased risk of delirium in hospitalized patients? (check one)
Not having family members present at the time of admission
Vision or hearing impairment
Repeated reorientation for patients with cognitive impairment
Early mobilization

A

Vision or hearing impairment

Delirium occurs in 11%–42% of hospitalized patients. Risk factors for delirium in hospitalized patients
include vision impairment, hearing impairment, dehydration, immobility, cognitive impairment, and sleep
deprivation. Repeated reorientation of patients with cognitive impairment, early mobilization, and
minimizing unnecessary noise or stimulation are all effective interventions for reducing the risk of delirium
in hospitalized patients. Not having family members available at the time of admission has no effect on
the incidence of delirium.

45
Q

A 70-year-old female becomes psychotic and risperidone (Risperdal) is prescribed. Which one of the following should be used to monitor the patient for adverse cardiac effects of this drug? (check one)
Serum sodium levels
Echocardiography
Nuclear stress testing
Lower-extremity venous duplex ultrasonography
Electrocardiography

A

Electrocardiography

Both typical and atypical antipsychotics can cause prolongation of the QTc interval, resulting in torsades
de pointes, ventricular tachycardia, and sudden death. The best way of monitoring the QTc interval is
electrocardiography.

46
Q

Which one of the following patients has an increased risk for autism spectrum disorder? (check one)
A 6-month-old who does not respond to his name
A 12-month-old who does not engage in pretend play
A 15-month-old who claps her hands when excited
An 18-month-old who does not point to objects of interest
A 24-month-old who looks at a parent’s face to see how to react in a situation

A

An 18-month-old who does not point to objects of interest

Autism spectrum disorder (ASD) is comprised of a range of neurodevelopment conditions that affect social communication and interaction and involve repetitive patterns of behavior, interests, or activities. Because early diagnosis and treatment can improve outcomes in affected patients and families, the American Academy of Pediatrics recommends universal screening for ASD with standardized autism-specific screening tools, such as the M-CHAT, at 18 and 24 months; regular developmental surveillance at all visits; and an appropriate response to family and caregiver concerns. An 18-month-old who does not point to show interest would be concerning for an increased risk for ASD, and further evaluation is warranted. A child who does not respond to his or her name by 12 months of age or engage in pretend play by 18–24 months of age would also be at risk for ASD. Clapping hands when excited is a normal milestone for a 15-month-old, and clapping is not considered a repetitive activity in this context. Looking at the parent’s face to see how to react, particularly in a new situation, is also a normal milestone for a 24-month-old.

47
Q

You are caring for a 21-year-old female with previously diagnosed bipolar II disorder, generalized anxiety disorder, attention-deficit/hyperactivity disorder, and insomnia. The patient presents for a same-day appointment with new symptoms of chills, excess sweating, flushing, and nausea of approximately 2 hours’ duration. The patient felt normal upon awakening, took methylphenidate (Ritalin), 5 mg with breakfast, and went to work. She began to feel shaky around lunchtime and took a second dose of methylphenidate, 5 mg. Thirty minutes later she began having agitation, chills, sweating, flushing, and nausea and had to leave work. Her current medications include the following:

Desvenlafaxine (Pristiq), 50 mg daily
Doxepin, 10 mg daily at bedtime
Methylphenidate, 5 mg twice daily
Ziprasidone (Geodon), 40 mg twice daily

An examination reveals an alert and anxious patient with damp skin, a temperature of 38.1°C (100.6°F), and a heart rate of 110 beats/min. The pupils are slightly dilated and briskly reactive. A neurologic examination reveals a mild tremor and hyperreflexia without clonus. (check one)
Discontinuing all current medications until her symptoms subside
Replacing methylphenidate with atomoxetine (Strattera)
Replacing methylphenidate with amphetamine salts such as dextroamphetamine/amphetamine (Adderall)
Symptomatic treatment with diphenhydramine (Benadryl Allergy)
Symptomatic treatment with ondansetron

A

Discontinuing all current medications until her symptoms subside

Serotonin syndrome is a serious condition that can be life-threatening. This patient is taking multiple serotonergic medications and displays features suggestive of serotonin syndrome. Signs and symptoms of serotonin syndrome include mental status changes (e.g., agitation, hallucinations, delirium, coma), autonomic instability (e.g., hyperthermia, tachycardia, labile blood pressure, diaphoresis, dizziness, flushing), neuromuscular changes (e.g., tremor, rigidity, hyperreflexia), and gastrointestinal symptoms (e.g., nausea, vomiting, diarrhea). A timely diagnosis and immediate discontinuation of serotonergic medications can help prevent worsening of the condition. Supportive care, sometimes in a hospital or intensive-care setting depending on severity, is the mainstay of treatment. Severe symptoms that necessitate hospital management include a temperature >38.5°C, confusion, delirium, and rigidity. Multiple classes of medications are associated with serotonin syndrome, including SSRIs/SNRIs, tricyclic antidepressants, antipsychotics, stimulants, triptans, and others. Changing to a different stimulant, or to a nonstimulant, would not help resolve serotonin syndrome, nor would symptomatic treatment with diphenhydramine or similar agents. Serotonin syndrome has been reported with 5-HT3 receptor antagonists such as ondansetron, particularly when used in combination with other serotonergic medications.

48
Q

You see a 30-year-old male for a routine health maintenance visit. The physical examination is normal, but he mentions that he has an overwhelming urge to keep checking and rechecking doors to make sure they are locked. He senses that something dangerous will happen if he does not do this. These thoughts and behaviors have become very distressing to him, and have started to interfere with his work and home life.

Which one of the following would be most appropriate at this time? (check one)
Alprazolam (Xanax)
Risperidone (Risperdal)
Sertraline (Zoloft)
Referral for psychodynamic psychotherapy

A

Sertraline (Zoloft)

Obsessive-compulsive disorder (OCD) affects 3% of the population and is characterized by intrusive thoughts (obsessions) and repetitive behaviors (compulsions). Obsessions are often religious, sexual, or violent in nature. They may include pathological doubting, preoccupation with contamination, concerns with symmetry, and a sense that something unpleasant or dangerous will happen if a particular ritual is not performed precisely. Typical compulsions include counting, checking, repeating, cleaning, and arranging behaviors. For the diagnosis to be made, these symptoms must be severe enough to cause marked distress or to impair functioning.

Most patients develop symptoms prior to age 35, and a large number of them keep these symptoms a secret. There is often a delay of 5–10 years before the illness comes to medical attention. With optimum treatment, 90% have moderate to marked improvement.

Treatment of OCD requires the integration of various approaches to maximize the outcome. Most patients experience substantial improvement using a combination of pharmacotherapy, particularly SSRIs, and cognitive behavioral therapy. Benzodiazepines such as alprazolam are capable of relieving generalized anxiety, but do not affect obsessions or compulsions. Antipsychotics such as risperidone may be added to an SSRI as second-line pharmacotherapy. Traditional psychodynamic psychotherapy is not effective for OCD.

49
Q

A 24-year-old female with a history of bulimia nervosa sees you for treatment of depression. She is currently receiving cognitive-behavioral therapy. You decide that she requires medication to treat her depression.
Which one of the following medications has been associated with an increased risk of seizures in patients with bulimia nervosa? (check one)
Bupropion (Wellbutrin)
Fluoxetine (Prozac)
Nortriptyline (Pamelor)
Sertraline (Zoloft)
Venlafaxine (Effexor XR)

A

Bupropion (Wellbutrin)

Antidepressants in every class (SSRIs, SNRIs, tricyclic antidepressants, and monoamine oxidase inhibitors) have been shown to reduce bulimic symptoms and can be used safely to treat depression, with the exception of bupropion. Bupropion use has been associated with an increased risk of seizures in patients with bulimia and an FDA warning limits its use.

50
Q

A 48-year-old male with schizophrenia presents for a new patient visit after recently relocating to your area. He has been stable on clozapine (Clozaril) for the past 15 years and asks you to refill his prescription. He has been told the earliest available appointment with a local psychiatric provider is in 3 months.

Under the Clozapine Risk Evaluation and Mitigation Strategy (REMS) program, which one of the following is required to prescribe clozapine to this patient? (check one)
A signed patient consent form
Serum clozapine levels
Creatinine levels
Neutrophil counts
Specialty training in psychiatry

A

Neutrophil counts

Clozapine is a highly effective antipsychotic medication, but its use is limited due to its association with severe drug-induced neutropenia, also referred to as agranulocytosis. Patients must be enrolled in the national Clozapine Risk Evaluation and Mitigation Strategy (REMS) program to receive treatment, and all prescribers and pharmacies must be certified by this program in order to dispense clozapine. The patient’s absolute neutrophil count must be submitted at least every 30 days, or more frequently as determined by stability in treatment. A signed patient consent form should be obtained but is not a part of the Clozapine REMS monitoring system. Monitoring serum clozapine levels and creatinine levels may be appropriate but is not part of the Clozapine REMS program. Family physicians can prescribe clozapine if registered and certified in the Clozapine REMS program, which includes passing a brief knowledge assessment, but specialty training in psychiatry is not required.

51
Q

Which one of the following is the most appropriate psychotherapy for patients with obsessive-compulsive disorder? (check one)
Traditional psychotherapy
Cognitive-behavioral therapy
Psychoanalysis
Psychodynamic therapy

A

Cognitive-behavioral therapy

Cognitive-behavioral therapy, specifically exposure and response prevention, is considered the most effective psychotherapy method (SOR A). There is no evidence for psychodynamic or “talk” therapy. Traditional psychotherapy and psychoanalysis are less effective than cognitive-behavioral therapy.

52
Q

You diagnose attention-deficit/hyperactivity disorder in a 4-year-old female. Which one of the following would be the most appropriate first-line treatment? (check one)
Dietary modification
Vitamin supplementation
Behavioral intervention
Atomoxetine (Strattera)
Methylphenidate (Ritalin)

A

Behavioral intervention

Attention-deficit/hyperactivity disorder in preschool children aged 4–5 years should be managed initially with behavioral intervention. Starting at age 6, pharmacologic medication such as atomoxetine or methylphenidate could be considered. There is no evidence of any benefit with dietary modification or vitamin supplementation.

53
Q

A 52-year-old male with a long-standing history of hypertension, COPD, type 2 diabetes mellitus, and bipolar disorder is brought to your office by his daughter because of a new onset of tremors. He is currently taking aspirin, hydrochlorothiazide, atenolol (Tenormin), glyburide (DiaBeta), lithium, inhaled albuterol, and inhaled tiotropium (Spiriva). Except for a recent episode of dehydration, his medication has worked well and no medication changes have been made within the past 2 years. On examination his heart rate is 52 beats/min and a neurologic examination reveals mild ataxia and coarse tremors. The remainder of the physical examination is normal.

Which one of the following is the most likely cause of his clinical findings? (check one)
Albuterol
Atenolol
Lithium
Tiotropium

A

Lithium

Lithium is a drug with a narrow therapeutic index and a low volume of distribution. Elderly patients are more likely to develop lithium toxicity due to their lower muscle mass and age-related decreased glomerular filtration rate (GFR). Chronic toxicity is more common than acute toxicity and is often precipitated by events causing volume depletion, such as vomiting, diarrhea, and acute gastroenteritis. Drugs that impact renal function or volume status, such as ACE inhibitors, NSAIDs, and diuretics, can also precipitate toxicity. Chronic toxicity often presents with signs and symptoms related to the gastrointestinal tract (nausea, vomiting, and diarrhea, which can further worsen toxicity), heart (arrhythmias and conduction delays), and central nervous system (coarse tremors, ataxia, agitation, and confusion). Albuterol and tiotropium both cause transient tremors and tachycardia, but are not associated with ataxia. Atenolol is associated with bradycardia, but not tremors or ataxia (SOR B).

54
Q

A 40-year-old white female sees you for the first time. When providing a history she describes several problems, including anxiety, insomnia, fatigue, persistent depressed mood, and low libido. These symptoms have been present for several years and are worse prior to menses, although they also occur to some degree during menses and throughout the month. Her menstrual periods are regular for the most part.

Based on this history, the most likely diagnosis is (check one)
premenstrual dysphoric disorder
menopause
dysthymia
anorexia nervosa
dementia

A

dysthymia

Psychological disorders, including anxiety, depression, and dysthymia, are frequently confused with premenstrual dysphoric disorder, and must be ruled out before initiating therapy. Symptoms are cyclic in true premenstrual dysphoric disorder. The most accurate way to make the diagnosis is to have the patient carefully record daily symptoms on a menstrual calendar for at least two cycles. Dysthymia consists of a pattern of ongoing, mild depressive symptoms that have been present for at least 2 years and are less severe than those of major depression, which is consistent with the findings in this case.

55
Q

An 84-year-old female with severe dementia due to Alzheimer’s disease is a resident of a long-term care facility. She has been hitting the staff while receiving personal care and recently had an altercation with another resident. Behavioral interventions have been unsuccessful in managing her symptoms and you suggest to the patient’s family that she be started on low-dose risperidone (Risperdal). They ask about appropriate use of the drug and the potential for side effects.

Which one of the following would be appropriate advice? (check one)
Extrapyramidal side effects are more common compared to typical antipsychotics
Dementia-related psychosis is an FDA-approved indication
No monitoring will be necessary
The risk of diabetes mellitus is decreased
The risk of mortality is increased

A

The risk of mortality is increased

Both typical and atypical antipsychotics increase the risk of mortality in patients with dementia. The FDA has a black box warning on these medications, including risperidone, about the increased risk of mortality in patients with dementia. Risperidone is not approved by the FDA for dementia-related psychosis. The typical antipsychotics are more commonly associated with extrapyramidal side effects. Diabetes mellitus and agranulocytosis are associated with the atypical antipsychotics, including risperidone. Periodic monitoring of serum glucose levels and CBCs is recommended.

56
Q

You suspect somatic symptom disorder in one of your patients. Supporting this diagnosis, the patient scores very high on the Somatic Symptom Scale–8 with “quite a bit” or “very much” back pain, chest pain, dizziness, low energy, headaches, stomach problems, and trouble sleeping. You have completed a thorough history and physical examination.

In addition to referral for cognitive behavioral therapy, which one of the following would be the most appropriate next step in patient care?

(check one)
Frequent, as-needed office visits
Frequent, regularly scheduled office visits
Infrequent, as-needed office visits
Infrequent, regularly scheduled office visits
Regularly scheduled annual examinations and as-needed office visits

A

Frequent, regularly scheduled office visits

In somatic symptom disorder, the primary feature is the patient’s concern with physical symptoms that have manifested through psychological or emotional distress and have no other medical explanation. The family physician should offer frequent, regularly scheduled visits for these patients.

The acronym CARE MD describes an approach to management of somatic symptom disorder:

  • Consultation with mental health professionals for cognitive behavioral therapy or mindfulness-based therapy (SOR B)
  • Assessment for other medical and psychiatric diseases
  • Regular short-interval follow-up visits to stress coping rather than cure and to reduce the overuse of health care visits such as the emergency department
  • Empathy demonstrated by listening to the patient while acknowledging and legitimizing the patient’s symptoms
  • Medical-psychiatric interface emphasizing the mind-body connection
  • Doing no harm by reassuring the patient while limiting diagnostic testing and referrals to subspecialists
57
Q

Intensive behavioral intervention has more benefit than other treatment modalities in treating children who have been diagnosed with
(check one)
attention-deficit/hyperactivity disorder
autism
obsessive-compulsive disorder
posttraumatic stress disorder

A

autism

The only evidence-based treatment that confers significant benefits to children with autism is intensive behavioral interventions, which should be initiated before 3 years of age. Attention-deficit/hyperactivity disorder can be treated with cognitive-behavioral therapy (CBT) but medication is often required. CBT is as effective, if not more effective, than medication for treating anxiety, depression, and trauma-related disorders.

58
Q

A 32-year-old female sees you for a health maintenance visit. She reports that she experiences severe anxiety when involved in social situations. She lives with her mother and dreads meeting unfamiliar people. At work she remains in her cubicle throughout the day and avoids staff parties. She has a history of alcoholism in remission. She has otherwise been in good health and a physical examination is normal.

Which one of the following would be first-line treatment for this patient? (check one)
Amitriptyline
Bupropion (Wellbutrin)
Escitalopram (Lexapro)
Lorazepam (Ativan)
Pregabalin (Lyrica)

A

Escitalopram (Lexapro)

Social anxiety disorder can be treated with psychotherapy, pharmacotherapy, or both. Several medications have been used for the treatment of social anxiety disorder. SSRIs are considered to be the first-line pharmacologic treatment. Response rates reported for the SNRI venlafaxine have been similar to those reported for SSRIs. Randomized trials have also supported the efficacy of benzodiazepines for social anxiety disorder, but they carry a risk of physiologic dependence and withdrawal symptoms and are not recommended for patients with coexisting depression or a history of substance abuse. Response rates to pregabalin have been lower than with SSRIs. Tricyclic antidepressants and bupropion are not considered to be useful in the treatment of social anxiety disorder.

59
Q

An 85-year-old female with advanced Alzheimer’s disease is brought to your office for treatment of agitation, aggressive behavior, and delusions. Behavioral and psychological interventions have had little success and the family is willing to try medications because they prefer to keep the patient at home.

Which one of the following would most likely help control this patient’s symptoms? (check one)
Alprazolam (Xanax)
Aripiprazole (Abilify)
Clozapine (Clozaril)
Haloperidol

A

Aripiprazole (Abilify)

Nonpharmacologic interventions are the first-line treatment for patients with behavioral and psychological symptoms of dementia. Antipsychotic medications can be prescribed for refractory cases but this is an off-label use. Both the patient and family should be aware that the use of atypical antipsychotics for behavioral symptoms of dementia is associated with increased mortality. Patients should be monitored for side effects and the medication should be discontinued if there is no evidence of symptom improvement after a month.

Typical antipsychotics such as haloperidol have significant side effects and would not be a good choice. Donepezil is initiated early in the course of Alzheimer’s disease to delay progression of the disease. Benzodiazepines are likely to cause significant side effects including sedation, increased confusion, and falls. Several of the antipsychotics, such as ziprasidone and clozapine, are ineffective. Results with olanzapine, quetiapine, and risperidone are inconsistent. Aripiprazole produces small reductions in behavioral and psychological symptoms of dementia, and it has the least adverse effects of the atypical antipsychotics.

60
Q

A 15-year-old female presents with a 3-month history of intermittent abdominal pain and headaches. She does not have any associated weight loss, fever, nausea, change in bowel habits, or other worrisome features. An examination is unremarkable. She does report being stressed at school and has a PHQ-2 score of 4.

Which one of the following would be most appropriate at this point? (check one)
Further evaluation for depression
Laboratory studies
Abdominal imaging
Citalopram (Celexa)
Fluoxetine (Prozac)

A

Further evaluation for depression

The U.S. Preventive Services Task Force recommends depression screening for all adolescents 12–18 years of age. Although this patient has abdominal pain, the history and physical examination suggest that depression may be playing a role in her somatic complaints. She had a positive initial depression screen on her PHQ-2. This is a brief screening tool, and a positive result merits further evaluation. The evaluation should include a full PHQ-A or a discussion with a qualified clinician. If the patient meets the criteria for major depressive disorder then she should receive treatment for her depression, which could include medication. Both fluoxetine and citalopram have been approved by the FDA to treat depression in this age group. She could also be referred for psychotherapy. Further laboratory studies and imaging may be appropriate at some point, but the most urgent need is to evaluate her positive depression screen.

61
Q

An 88-year-old female hospice patient is experiencing significant agitation and delirium. After attempting conservative treatments and ruling out reversible causes of her symptoms, which one of the following pharmacologic agents would be most appropriate? (check one)
Alprazolam (Xanax)
Amitriptyline
Diphenhydramine (Benadryl)
Risperidone (Risperdal)

A

Risperidone (Risperdal)

Agitation and delirium are common end-of-life symptoms. It is important to assess for treatable causes,
including constipation, urinary retention, uncontrolled pain, and adverse medication effects. The
antipsychotic medication risperidone is effective for treating agitation and nausea at this stage, but dosing
is much lower than when this medication is used for psychiatric disorders. Benzodiazepines can provoke
increased agitation and should be used with caution; however, they can be useful for treating significant
end-of-life anxiety. Generally, a longer-acting form such as lorazepam would be a better choice than
short-acting alprazolam. Amitriptyline and diphenhydramine can both cause urinary retention, potentially
leading to delirium and agitation.

62
Q

Your patient, who is overweight and says she has struggled with bulimia in the past, asks for your advice on strategies to help prevent obesity and eating disorders in her 12-year-old daughter. Which one of the following strategies should be incorporated? (check one)
Implementing a diet of moderate calorie restriction for healthy adolescents and adults 1 week per month
Having more frequent discussions regarding weight control and healthy eating
Limiting home-prepared dinners to 1–2 times per week
Eating meals together as a family ³7 times per week
Watching television during mealtimes

A

Eating meals together as a family ³7 times per week

A higher frequency of family meals is associated with improved dietary quality, as evidenced by increased consumption of fruits, vegetables, and grains. Eating meals together as a family on most days or every day is protective against purging and binge eating, as well as frequent dieting, which is a risk factor for both obesity and eating disorders. Parental talk about weight revolving around their children or their own dieting is linked to becoming overweight. Meals that are home-prepared and undistracted are also beneficial to maintaining healthy weight and attitudes toward food.

63
Q

A 75-year-old male presents with weight loss, fatigue, and sleep disturbance. He also has a 6-week history of feeling depressed and not enjoying life anymore. He is not suicidal. After an appropriate evaluation to rule out other causes, you diagnose major depressive disorder.

Which one of the following agents is indicated as first-line pharmacotherapy for this patient? (check one)
Aripiprazole (Abilify)
Dextroamphetamine/amphetamine (Adderall)
Nortriptyline (Pamelor)
Sertraline (Zoloft)
Venlafaxine (Effexor XR)

A

Sertraline (Zoloft)

Because of their favorable side-effect profile and low cost, SSRIs are the first-line agents for late-life depression. SNRIs may be used as second-line agents when remission is not obtained with an SSRI. Tricyclic antidepressants work as well as SSRIs and may be considered in recalcitrant cases, but side effects may be troublesome, especially in this age group. The use of stimulants in depressed older adults has not been well studied. Second-generation antipsychotic agents may be used as an add-on to an SSRI or SNRI medication when the depression is resistant. Side effects and long-term safety may be issues with this approach.

64
Q

A 14-year-old female with mild depression sees you for follow-up. After her last visit she began a trial of psychotherapy but her mother asks about additional treatment options. The daughter has no other significant past medical history.

Which one of the following is recommended as first-line pharmacotherapy for this patient? (check one)
Citalopram (Celexa)
Fluoxetine (Prozac)
Mirtazapine (Remeron)
Paroxetine (Paxil)
Venlafaxine (Effexor XR)

A

Fluoxetine (Prozac)

Expert panel guideline recommendations suggest fluoxetine should be the first-line pharmacotherapy option for adolescents after a trial of psychotherapy. The patient should be monitored weekly for side effects for a month after starting fluoxetine. If fluoxetine is ineffective, sertraline and citalopram are recommended as alternatives. Venlafaxine should be avoided in adolescents because it is associated with a statistically increased risk of suicidal behavior or ideation.

65
Q

A patient is brought to your office by his spouse because of anger issues over the last 6 weeks that are affecting his marriage and his work. He tells you he has started running 5 miles per day, but is sleeping poorly, and that his mind has been racing during this period. He recently went on a shopping spree and reached the maximum spending limit on his credit card. He has been using alcohol to calm down. He refuses to see a psychiatrist.

The most appropriate medication to initiate at this time is (check one)
bupropion (Wellbutrin)
lithium
lorazepam (Ativan)
sertraline (Zoloft)
trazodone (Oleptro)

A

lithium

This patient is experiencing a manic or hypomanic episode. Therapeutic options include lithium,
anticonvulsants such as divalproex, and antipsychotic medications such as olanzapine. Benzodiazepines
such as lorazepam may be of minimal benefit. SSRIs such as sertraline can aggravate mania. Bupropion
would help treat an associated depression and trazodone could possibly help the patient sleep, but these
medications are ineffective for treating a manic/hypomanic episode.

66
Q

A 46-year-old female with bipolar I disorder in remission seeks your advice regarding discontinuation of her medications, which include sertraline (Zoloft), 50 mg daily; quetiapine (Seroquel), 200 mg nightly; and hydroxyzine, 25 mg taken occasionally as needed for anxiety.

Which one of the following would you advise this patient? (check one)
Continue all current medications without change
Discontinue all medications except for as-needed use of hydroxyzine
Discontinue only quetiapine
Discontinue either sertraline or quetiapine
Discontinue quetiapine and start an alternative mood stabilizer

A

Continue all current medications without change

Accepted mood stabilizers used for maintenance therapy in patients with bipolar disorder include lithium, valproate, lamotrigine, and some atypical antipsychotics such as olanzapine, quetiapine, and risperidone. The atypical antipsychotics are associated with weight gain and adverse metabolic changes. Annual testing for diabetes mellitus is recommended. Long-term maintenance therapy with a mood stabilizer is recommended in patients with bipolar I disorder due to the high risk of recurrent mania. Monotherapy with antidepressants is contraindicated. Although the typical antipsychotics may cause QT
prolongation, atypical antipsychotics such as quetiapine do so much less frequently, and most SSRIs do not cause this. Coadministration with sertraline is not contraindicated.

67
Q

A 36-month-old male has persistent deficits in social communication and interaction across multiple contexts. He displays restricted and repetitive patterns of behavior, interest, and activities.

According to the DSM-5 , which one of the following is the most appropriate diagnosis? (check one)
Asperger syndrome
Autism spectrum disorder
Autistic disorder
Childhood disintegrative disorder
Pervasive developmental disorder not otherwise specified

A

Autism spectrum disorder

Key diagnostic features of autism spectrum disorder include deficits in social communication and interaction across multiple contexts and restricted, repetitive patterns of behavior, interests, or activities. The DSM-5 , which came out in 2013, created an umbrella diagnosis of autism spectrum disorder to consolidate four previously separate disorders: autistic disorder, Asperger’s disorder, childhood disintegrative disorder, and pervasive developmental disorder not otherwise specified. Any individuals with a previous diagnosis of one of these disorders should now be given a diagnosis of autism spectrum disorder.

68
Q

While making rounds at a nursing home you see a 70-year-old female with dementia. The staff tells you that she has recently developed serious aggressive behaviors that include lashing out physically at caregivers on a regular basis. Nonpharmacologic interventions have not curbed her violent outbursts. Your evaluation does not reveal any treatable underlying conditions.

After a conversation about risks and benefits with her family and the nursing home staff, which one of the following would you recommend for this patient? (check one)
Diphenhydramine (Benadryl)
Aripiprazole (Abilify)
Clonazepam (Klonopin)
Mirtazapine (Remeron)
Ziprasidone (Geodon)

A

Aripiprazole (Abilify)

Although the FDA has not approved the use of antipsychotics for aggressive behavior associated with dementia, they are often used to treat refractory behavioral and psychological symptoms of dementia. Their off-label use should be considered only when nonpharmacologic therapies are ineffective and the behaviors pose a risk of harm to the patient or others (SOR C), and the drug should be discontinued if there is no evidence of symptom improvement (SOR A). In a meta-analysis of three atypical antipsychotics, only aripiprazole showed small average reductions in behavioral and psychological symptoms of dementia. Olanzapine has demonstrated inconsistent results and ziprasidone is ineffective. Diphenhydramine is an anticholinergic agent and could exacerbate behaviors. Mirtazapine is indicated for depression. The American Geriatrics Society recommends against the use of benzodiazepines in older adults as a first choice for insomnia, agitation, or delirium.

69
Q

A 10-year-old female is brought to your office by her parents on the recommendation of the school counselor. Her parents report that for at least the past year, both at home and at school, their daughter often loses her temper, is easily annoyed, and is “very touchy.” She is sullen and angry, arguing frequently with her parents and teachers. At school she irritates and annoys classmates, then blames them when they react negatively toward her. She also disrupts the classroom by refusing to comply with classroom rules and expectations or with the teacher’s authority.

This child’s behavior is most consistent with which one of the following? (check one)
Attention-deficit/hyperactivity disorder
Bipolar disorder
Conduct disorder
Intermittent explosive disorder
Oppositional defiant disorder

A

Oppositional defiant disorder

This child presents with oppositional defiant disorder (ODD). To meet the DSM-5 criteria for ODD, the child must demonstrate at least four symptoms from any of the following categories: angry/irritable mood (often loses temper, is often touchy or easily annoyed, is often angry and resentful), argumentative/defiant behavior (often argues with authority figures or with adults, often actively defies or refuses to comply with requests from authority figures, often deliberately annoys others, often blames others for his/her mistakes or misbehavior), and vindictiveness (has been spiteful or vindictive at least twice within the past 6 months). These behaviors must be directed toward at least one person other than a sibling.

Behavioral problems associated with conduct disorder are more severe, including aggression toward animals or other persons, destruction of property, and a pattern of theft or deceit. The person’s conduct frequently leads to conflict with authority figures.

Attention-deficit/hyperactivity disorder involves difficulty in following rules, struggles with authority figures, and possibly annoying others. In ODD, however, defiance of authority figures occurs in settings other than those where sustained attention or sitting quietly is required.

Bipolar disorder can include irritability and negative affect but the argumentative, defiant behavior or vindictiveness that occur in ODD do not routinely occur in mood disorders.

Intermittent explosive disorder involves repeated, sudden episodes of impulsive, aggressive, violent behavior or angry verbal outbursts in which the person reacts grossly out of proportion to the situation. Road rage, domestic abuse, throwing or breaking objects, or other temper tantrums may be signs of intermittent explosive disorder. Serious aggression toward others does not occur in ODD.

70
Q

A 26-year-old male graduate student presents with concerns about having attention-deficit disorder (ADD). He reports inattentiveness and a lack of concentration for the last 2–3 years, resulting in poorer academic performance than when he was an undergraduate. He says his wife also complains of his lack of focus and attentiveness when attending to household responsibilities. He does not recall having similar symptoms in grade school or high school.

You explain that in order to diagnose adult ADD (check one)
evidence of symptoms of ADD must be present before age 6
evidence of symptoms of ADD must be present before age 12
a therapeutic trial of a short-acting dextroamphetamine/amphetamine combination (Adderall) is indicated
a therapeutic trial of an SNRI such as atomoxetine (Strattera) is indicated

A

evidence of symptoms of ADD must be present before age 12

Diagnosing attention-deficit disorder in adults requires symptoms that interfere with social, academic, or occupational functioning and are present in more than one setting. DSM-5 states that a history of symptoms before age 12 is required for the diagnosis. DSM-IVspecified that symptoms must have been present before age 7.

71
Q

An 85-year-old female is admitted to the hospital for surgery and develops confusion postoperatively. The patient is a widow and lived independently prior to admission. Her daughter says the confusion is atypical for her mother. She does not have a history of memory loss, forgetfulness, or confusion prior to admission.

Which one of the following cognitive assessment tests should be used to assess her acute change in mental status? (check one)
Confusion Assessment Method (CAM)
Mini-Cog
Mini-Mental State Examination (MMSE)
Montreal Cognitive Assessment test (MoCA)
Saint Louis University Mental Status exam (SLUMS)

A

Confusion Assessment Method (CAM)

The patient is experiencing an acute cognitive change from baseline, indicating possible delirium. The Confusion Assessment Method (CAM) is a delirium diagnosis tool useful for evaluating acute cognitive changes. The other tests listed, including the Mini-Mental State Examination, Mini-Cog, Montreal Cognitive Assessment, and Saint Louis Mental Status exam, test chronic baseline cognitive function and are not designed to test for acute changes.

72
Q

A 77-year-old female with widely metastatic lung cancer is seen in her home with increased confusion, hallucinations, and combative behavior for the past 2 days. She has not slept and has a very poor appetite.

Which one of the following should be the first step in the management of this patient? (check one)
Place her in gentle restraints for safet
Determine and treat the underlying cause of the behavior change
Start diphenhydramine (Benadryl) at bedtime for sleep
Start mirtazapine (Remeron) at bedtime for sleep and appetite
Start lorazepam (Ativan) as needed for agitation

A

Determine and treat the underlying cause of the behavior change

This patient has hyperactive delirium. The first step in management is to determine and treat the underlying cause if possible. There are multiple causes of delirium such as medications, infections, metabolic abnormalities, and underlying diseases. The first step in treatment is behavioral management with strategies to orient the patient. Haloperidol or antipsychotics may be used if the patient is at risk of harm. Lorazepam and anticholinergics should both be avoided, as they can worsen delirium. Restraints can also worsen the agitation and should not be used. Mirtazapine is an antidepressant and is not used in the treatment of delirium.

73
Q

A 43-year-old female presents with an 8-month history of posttraumatic stress disorder following a motor vehicle accident that severely injured her and a friend. She has had a positive response to counseling and SSRI treatment but continues to have sleep disturbances and nightmares.

Which one of the following medications is most likely to decrease the frequency of her nightmares? (check one)
Clonazepam (Klonopin)
Divalproex (Depakote)
Prazosin (Minipress)
Propranolol

A

Prazosin (Minipress)

Prazosin is an α-adrenergic receptor antagonist and is recommended for the treatment of nightmares in posttraumatic stress disorder (SOR A). It is thought to reduce sympathetic outflow in the brain. Although clonidine may be tried, evidence of its effectiveness is sparse (SOR C). Clonazepam, propranolol, and divalproex have not been recommended.

74
Q

A 24-year-old male complains of feeling on edge all of the time. For the past 2 years he has had difficulty controlling his worrying about work, school, and relationships. He has had more difficulty concentrating at work and school, is more irritable, and has difficulty staying asleep all night. He drinks alcohol moderately and does not use drugs. You recommend regular exercise and refer him to a therapist for cognitive-behavioral therapy to help manage his symptoms.

Which one of the following would be first-line medical therapy for this patient? (check one)
Bupropion (Wellbutrin)
Fluoxetine (Prozac)
Lorazepam (Ativan)
Methylphenidate (Ritalin, Concerta)
Quetiapine (Seroquel)

A

Fluoxetine (Prozac)

Though symptoms of generalized anxiety disorder (GAD) overlap with other psychiatric and medical conditions, the case presented is most consistent with GAD. SSRIs are first-line therapy for GAD (SOR B). Benzodiazepines such as lorazepam can improve anxiety-related symptoms, but due to the side effects and addiction potential they are recommended for short-term use (SOR B). Bupropion is approved for the treatment of depression but is not used to treat GAD. Quetiapine may be considered as second-line therapy for GAD (SOR B). Methylphenidate is first-line therapy for attention-deficit/hyperactivity disorder but is not indicated to treat GAD. Psychotherapy, especially cognitive-behavioral therapy, is also first-line treatment for GAD (SOR A), and exercise can also improve symptoms (SOR B).

75
Q

A patient is brought to the emergency department by his wife due to acute anxiety, jittery movements, confusion, vomiting, and fever, all of which started without warning a few hours ago. The wife reports that he has had a cough and upper respiratory symptoms recently, and he took an over-the-counter medication with dextromethorphan this morning. His usual medications include fluoxetine (Prozac), 30 mg daily for depression, and methylphenidate (Metadate CD), 50 mg daily for attention-deficit/hyperactivity disorder. She checked his medication bottles and does not think he has taken extra doses.

His vital signs include a blood pressure of 160/95 mm Hg, a heart rate of 116 beats/min, a respiratory rate of 25/min, a temperature of 38.5°C (101.3°F), and an oxygen saturation of 98% on room air. A physical examination is remarkable for restlessness, anxiety, diaphoresis, and inducible clonus most prominent in the lower extremities. His lungs are clear and his neck is supple. He is alert and oriented to self only.

Laboratory studies reveal a WBC count of 14,000/mm3 (N 4500–11,000) and a serum bicarbonate level of 20 mEq/L (N 23–30), and a urine drug screen is positive for amphetamines only. A COVID-19 polymerase chain reaction test is negative.

The most likely cause of his symptoms is: (check one)
an overdose of methylphenidate
an infectious process
malignant hyperthermia
serotonin syndrome

A

serotonin syndrome

Symptoms of serotonin syndrome range from mild to life-threatening and typically appear minutes to hours after ingestion of serotonergic medications. SSRIs are the most commonly associated class of medication due to their widespread use. The Hunter Serotonin Toxicity Criteria are the most commonly used diagnostic tool. This patient has a history of serotonergic medication use, signs of inducible clonus, agitation, and diaphoresis, as well as hyperthermia. It is likely that the addition of dextromethorphan precipitated this episode. This patient’s history does not suggest an overdose of methylphenidate, and there is little evidence in this scenario for a serious infectious process. Malignant hyperthermia generally appears over a longer period of time and does not typically induce clonus. There are few, if any, choices for medication therapy of concomitant attention-deficit/hyperactivity disorder and depression that do not increase the risk of serotonin syndrome, so patients on these regimens should be educated about the symptoms of serotonin syndrome and common causative agents.

76
Q

A 34-year-old female has posttraumatic stress disorder that started 6 months ago after she was a victim of an armed robbery. She also has nightmares, anxiety, hypervigilance, and some paranoia and has been reexperiencing the robbery. She has been receiving psychotherapy but would now like to start a medication.

Which one of the following would be the most appropriate medication to start? (check one)
Bupropion (Wellbutrin)
Buspirone
Lorazepam (Ativan)
Risperidone (Risperdal)
Venlafaxine (Effexor XR)

A

Venlafaxine (Effexor XR)

It is important for patients with posttraumatic stress disorder (PTSD) to receive psychotherapy. For those
who continue to have symptoms it is appropriate to initiate medications. SSRIs and venlafaxine are
considered first-line medications for the treatment of PTSD. Bupropion and buspirone have not been found
to be effective treatments. The use of benzodiazepine medications is not recommended because of the high
risk of misuse. Antipsychotic medications would be appropriate in patients with disabling symptoms and
behaviors that do not respond to psychological or drug treatment.

77
Q

You suspect that a 28-year-old female patient may have a somatic symptom disorder, specifically a conversion disorder. Which one of the following is the most appropriate pharmacologic treatment of this disorder? (check one)
Bupropion (Wellbutrin)
Clozapine
Selegiline (Eldepryl)
Sertraline (Zoloft)
Topiramate (Topamax)

A

Sertraline (Zoloft)

Somatic symptom disorders account for approximately 5% of primary care visits. Effective pharmacologic
treatment includes sertraline and other SSRI-based therapy in addition to cognitive-behavioral therapy
(SOR B). Bupropion, monoamine oxidase inhibitors such as selegiline, anticonvulsants such as topiramate,
and antipsychotics such as clozapine are ineffective.

78
Q

An 82-year-old female in your palliative care service who has stage 4 breast cancer is experiencing frequent episodes of delirium. Her pain is well controlled on long-acting oral opioid therapy. Additionally, no other reversible causes of delirium are noted. Her delirium is not responding to conservative measures, and her family asks if there are any medications that can effectively manage her symptoms.

Which one of the following should you recommend? (check one)
Alprazolam (Xanax)
Diazepam (Valium)
Melatonin
Risperidone (Risperdal)

A

Risperidone (Risperdal)

The first step in managing delirium in end-of-life care is to assess for any reversible or treatable causes, including uncontrolled pain, constipation, urinary retention, infections (e.g., urinary tract infections), and medication side effects. Antipsychotic medications, such as haloperidol and risperidone, are recommended if conservative measures fail to control the symptoms of delirium. Benzodiazepines should be used with caution as they can worsen delirium, especially in older patients. Melatonin is not indicated in the management of delirium.

79
Q

A first-time mother brings her 12-month-old to your office for a well child check. She and the child’s father are both your patients as well. Upon routine screening for anemia, you discover the infant has microcytic anemia with elevated red cell distribution width. You ask the parent about the child’s diet. She looks down at the floor, exhibiting poor eye contact and a flat affect, and responds that she does not shop or cook often. She says the child is happy when eating just applesauce and milk. The mother reports that she has been unmotivated and crying frequently.

In addition to checking the child’s lead level and starting iron supplementation, which one of the following would be the most important next step to help this dyad? (check one)
Providing reassurance and validation for the mother
Educating the mother about nutrition and the importance of iron-rich foods for the child
Notifying the child’s father
Evaluating and treating the mother for postpartum depression
Referring the mother to a psychiatrist

A

Evaluating and treating the mother for postpartum depression

Postpartum depression is common and patients who have given birth should be screened for a minimum of 1 year. A mother who knows what to feed an infant may have trouble executing it because of severe postpartum depression, leading to poor infant feeding practices. In this case, the provider will be more likely to successfully treat the child’s anemia by treating the mother’s postpartum depression.§While mothers with depression often need encouragement and support, false reassurance is paternalistic and potentially harmful. Education of the mother is sometimes useful, but studies have demonstrated that the more likely barrier to implementing her knowledge is not a need for education on infant nutrition, but rather severe depression or other psychosocial barriers. It is ideal to involve all caretakers in efforts to support a child’s health, but a number of steps need to be taken prior to calling the father, including screening for domestic violence, checking on HIPAA consents, and asking about custody. Referring the mother to a psychiatrist may ultimately be helpful but puts unnecessary barriers in place for the testing and treatment of postpartum depression.

80
Q

A 27-year-old white female with a history of mania sees you because of polyuria and increased thirst over the past month. She has taken lithium, 1800 mg daily, for 3 years and her mania is well controlled. She has not lost weight and there is no family history of her current problem. There are no orthostatic blood pressure changes.

Laboratory Findings

Serum sodium 145 mEq/L (N 135–145)
Serum potassium 4.5 mEq/L (N 3.5–5.0)
Serum glucose 92 mg/dL
Serum creatinine 0.9 mg/dL (N 0.6–1.5)
Serum lithium 1.38 mEq/L (therapeutic range 0.5–1.5)
Urine volume 6.85 L/24 hr
Urine osmolality 161 mOsm/kg H2O

There is no significant change in urine osmolality in response to the administration of vasopressin. Which one of the following is the most likely cause of this patient’s problem? (check one)
Drug-induced nephrogenic diabetes insipidus
Borderline diabetes mellitus
Panhypopituitarism
Psychogenic water drinking

A

Drug-induced nephrogenic diabetes insipidus

Polyuria occurs in 20%–70% of patients on long-term lithium therapy, even when plasma lithium levels
are in the therapeutic range. This is a result of impaired renal concentrating ability that is resistant to
vasopressin (nephrogenic diabetes insipidus). Inappropriate antidiuretic hormone secretion causes
hyponatremia and fluid retention. The diuresis associated with diabetes mellitus is a result of the osmotic
effect of increased serum glucose, which is not present in this case. Patients with hypothalamic or pituitary
injuries may develop central diabetes insipidus, which responds to exogenous vasopressin. Psychogenic
water drinking occurs in psychiatric patients, but would not be expected to cause impairment of renal
concentration or hypernatremia.

81
Q

A recently divorced 47-year-old male comes to your office appearing disheveled, with the smell of alcohol on his breath. His Patient Health Questionnaire–9 (PHQ-9) score today is 20, and his last PHQ-9 score was 7. He has a history of depression and is currently taking citalopram (Celexa). The patient is tearful during the encounter and admits to thinking the world would be better without him in it. He does not have a weapon with him but keeps a gun in an unlocked drawer in his nightstand.

Which one of the following would be most appropriate at this point? (check one)
Avoiding direct inquiry about suicide
Calling 911
Crisis planning
Creating a suicide prevention contract
Withholding psychogenic medications

A

Crisis planning

Crisis planning is recommended for patients presenting with suicidal ideation (SOR B). By identifying social support, local resources, and counseling services, suicidal ideation and days spent in the hospital can be reduced. Direct inquiry about suicide is recommended to better evaluate and treat suicidal patients with more favorable outcomes (SOR B). Though calling 911 may be appropriate for transportation for inpatient therapy if involuntary treatment is recommended, further assessment is needed in this case. Suicide prevention contracts do not effectively prevent suicide (SOR B). Psychogenic medications should not be withheld when treating a patient with suicidal ideation. Evidence has shown that the combination of pharmacotherapy and psychotherapy is most effective (SOR C).

82
Q

A 35-year-old gravida 2 para 2 reports diminished sexual arousal since initiating antidepressant therapy with sertraline (Zoloft). She has normal menstrual cycles and does not have any other symptoms.

Which one of the following would you recommend? (check one)
Black cohosh, 40 mg daily
Bupropion (Wellbutrin), 150 mg twice daily
Ethinyl estradiol, 0.5 mg daily
Ospemifene (Osphena), 60 mg daily
A testosterone patch, 300 μg applied twice weekly

A

Bupropion (Wellbutrin), 150 mg twice daily

Bupropion can improve antidepressant-related sexual arousal dysfunction (SOR B). Black cohosh is
considered a safe alternative for treating menopausal vasomotor symptoms, but not for treating sexual
arousal dysfunction in women who are premenopausal. Ethinyl estradiol may be taken to improve sexual
dysfunction related to menopausal symptoms. Vaginal estrogen therapy is recommended over oral estrogen
when vaginal dryness is the primary symptom. Ospemifene is indicated for dyspareunia related to vulvar
and vaginal atrophy due to menopause. Testosterone has proven to be effective for treating menopause-
9
related low sexual desire but the evidence is limited due to the lack of long-term data. The Endocrine
Society recommends consideration of a 3- to 6-month course of testosterone specifically for
postmenopausal women with low sexual desire.